Đến nội dung

phanquockhanh

phanquockhanh

Đăng ký: 27-12-2012
Offline Đăng nhập: 07-05-2016 - 09:23
***--

#508085 Giải phương trình sau: $4x^2+14x+4=(8x+5)\sqrt{2x+1}$

Gửi bởi phanquockhanh trong 20-06-2014 - 20:43

Giải phương trình sau:

$4x^2+14x+4=(8x+5)\sqrt{2x+1}$




#448950 $\left\{\begin{matrix}u_1= 1 \\...

Gửi bởi phanquockhanh trong 08-09-2013 - 22:05

Ta có: 

$u_1=1$

$u_2=u_1 +1$

$u_3= u_2 +2$

...

$u_n= u_{n-1} +n-1$

Cộng vế theo vế ,ta được

$u_n=u_1 +1+2+...+...+n-1=1+\dfrac{(n-1)n}{2}=\frac{n^2-n+2}{2}$

Từ đó,ta dễ dàng tìm được: $u_{2013}$

====

P/s: Từ công thức tổng quát trên ta có thể tìm được: $\lim_{n\rightarrow +\infty }\frac{u_n}{u_{n+1}}$ (Đề thi học sinh giỏi Hà Nội )




#446657 Bất đẳng thức chuẩn bị cho kì thi THPTQG 2015-2016

Gửi bởi phanquockhanh trong 31-08-2013 - 20:51

Bài 58: Cho $\ x, y>0$ thỏa mãn $\ x+y \leq 1$. Tìm GTNN của biểu thức:
 
$\ P=\sqrt{4x^2+\dfrac{1}{x^2}}+\sqrt{4y^2+\dfrac{1}{y^2}}-\dfrac{x}{x^2+1}-\dfrac{y}{y^2+1}$
(Đề thi thử Đại học lần 2 - Chuyên Lê Qúy Đôn, Vũng Tàu)



#443945 Bất đẳng thức chuẩn bị cho kì thi THPTQG 2015-2016

Gửi bởi phanquockhanh trong 18-08-2013 - 21:10

Bài 21: Cho $a,b,c $ là các số thực dương thỏa mãn $abc=1$.Tìm min của biểu thức:

$P=\dfrac{1}{2a+1}+\dfrac{1}{2b+1}+\dfrac{2}{(2c+1)\sqrt{6c+3}}$




#437775 Nếu $a\equiv b (mod p^n)$ thì $a^p\equiv b^p( mod p^...

Gửi bởi phanquockhanh trong 24-07-2013 - 14:59

Cho p là số nguyên tố ,n là số nguyên dương.Chứng minh rằng:

Nếu $a\equiv b (mod p^n)$ thì $a^p\equiv b^p( mod p^{n+1})$ 

 




#437609 Giải phương trình $2^{x^2+3cosx}-2^{x^2+4cos^3x}=7co...

Gửi bởi phanquockhanh trong 23-07-2013 - 21:19

Giải phương trình $$2^{x^2+3cosx}-2^{x^2+4cos^3x}=7cos3x$$ (1)

Bài giải:

$$(1)\Leftrightarrow 2^{x^2 +3cosx}+7(3cosx+x^2)=2^{x^2+4cos^3x}+7(4cos^3x +x^2) \,\, (2)$$

Xét hàm số: $f(t)=2^t+7t $ trên R.

Ta có $$f'(t)=2^tln2+7> 0;\forall t \in R$$

$\Rightarrow f(t)$ đồng biến trên R

Khi đó: $$(2)\Leftrightarrow f(x^2+3cosx)=f(4cos^3x+x^2)$$

$$\Leftrightarrow 3cosx =4cos^3x$$

$$ \Leftrightarrow cos3x =0 \Leftrightarrow x=\dfrac{\pi}{6}+k\dfrac{\pi}{3};k \in Z$$

Vậy:

$$x=\dfrac{\pi}{6}+k\dfrac{\pi}{3};k \in Z$$




#437602 Tổng hợp các bài BĐT

Gửi bởi phanquockhanh trong 23-07-2013 - 21:11

Bài 100:(b2stfs)

Cho a,b,c >0 thoả mãn a+b+c=3.Tìm GTNN của:
P=$\sum \frac{b\sqrt{b}}{\sqrt{2a+b+c}}$

 

Bài giải:(Sagittrius912)

Ta có

$P^{2}.((2a+b+c)+(2b+c+a)+(2c+a+b))= \sum \frac{b\sqrt{b}}{\sqrt{2a+b+c}}.\sum \frac{b\sqrt{b}}{\sqrt{2a+b+c}}.((2a+b+c)+(2b+c+a)+(2c+a+b))\geq (a+b+c)^{3}$

(bdt Holderr)

$\Rightarrow P^{2}\geq \frac{(a+b+c)^{2}}{4}\Rightarrow P\geq \frac{3}{2}$

Vậy min P=3/2 khi a=b=c=1

 

 

Bài 101:(b2stts)

Cho a,b,c>0 thoả mãn:a+b+c=$\frac{1}{2}$,Tìm GTLN:
$\sum \sqrt{\frac{(a+b)(b+c)}{(a+b)(b+c)+a+c}}$

Bài giải:(Mazacar)

Đặt $x=a+b;y=b+c;z=c+a$
Có:
$P=\sum \sqrt{\frac{xy}{xy+z}}=\sum \sqrt{\frac{xy}{(x+z)(z+y)}}\leq\frac{1}{2}\sum (\frac{x}{x+z}+\frac{y}{z+y})=\frac{3}{2}\blacksquare$
$"=" <=>x=y=z=\frac{1}{6}$

 

 

Bài 102:(Ispectorgadget)

Cho $a,b,c$ thực dương. Chứng minh:
$$1<\frac{a}{a+b}+\frac{b}{b+c}+\frac{c}{c+a}+\sqrt{\frac{2abc}{(a+b)(b+c)(a+c)}} \le 2$$
Liệu có thể thay số 1 trong bất đẳng thức trên bởi một số lớn hơn được không?

 

 

Bài giải:

(WhjteShadow)$\bullet$ Vế trái:
Do $a,b,c$ là các số thực dương nên $a+b<a+b+c\\ b+c<a+b+c\\ c+a<a+b+c$
Từ đó ta có:
$$\frac{a}{a+b}+\frac{b}{b+c}+\frac{c}{c+a}>\frac{a+b+c}{a+b+c}=1$$
$$\sqrt{\frac{2abc}{(a+b)(b+c)(a+c)}}>0$$ 
Nên ta có:
$$1<\frac{a}{a+b}+\frac{b}{b+c}+\frac{c}{c+a}+\sqrt{\frac{2abc}{(a+b)(b+c)(a+c)}}$$
Ta không thể thay $1$ bằng số khác ( Thật vậy thử với $a,b\to 0$ )
$\bullet$ Vế phải:
Đặt $\frac{b}{a}=x,\frac{c}{b}=y,\frac{a}{c}=z$ ($x,y,z>0,xyz=1$) ta cần chứng minh:
$$\frac{1}{x+1}+\frac{1}{y+1}+\frac{1}{z+1}+\sqrt{\frac{2}{(x+1)(y+1)(z+1)}}\leq 2$$
Quy đồng và đưa bất đẳng thức về the0 ngôn ngữ $p,q,r$ với chú ý $r=1$ ta có bất đẳng thức tương đương:
$$\frac{3+2p+q}{2+p+q}+\sqrt{\frac{2}{2+p+q}}\leq 2$$
$$\Leftrightarrow \sqrt{\frac{2}{2+p+q}}\leq \frac{1+q}{2+p+q}$$
$$\Leftrightarrow 2(2+p+q)\leq (1+q)^2$$
$$\Leftrightarrow 3+2p\leq q^2$$
Điều này luôn đúng do $q^2\geq 3pr=3q\geq 2p+3$
Kết thúc chứng minh. Dấu đẳng thức xảy ra tại $a=b=c$ $\square$

 

 

Bài 103:

Cho $x,y,z >0$ thoả mãn: $\sum x\geq \sum \frac{1}{x}$.Chứng minh rằng: $$\sum x\geq \frac{3}{x+y+z}+\frac{2}{xyz}$$
___
NLT: Chú ý tiêu đề bài viết!

Bài giải:

$x+y+x \geq \frac{1}{x}+\frac{1}{y}+\frac{1}{z} \geq \frac{9}{x+y+z} $
$\Rightarrow x+y+z \geq 3 $
$\Rightarrow (x+y+z)^2 = \frac{(x+y+z)^2}{3}+\frac{2(x+y+z)^2}{3} \geq 3 + \frac{2(\frac{1}{x}+\frac{1}{y}+\frac{1}{z})^2}{3} \geq 3 + 2( \frac{1}{xy}+\frac{1}{yz}+\frac{1}{zx})$
đpcm

 

Bài 104:
cho a,b,c >0 thoả mãn:a+b+c=6.CMR:
$\sum \frac{a}{\sqrt{(b+2)(b^{2}-b+2)}}\geq \frac{3}{2}$

 

Bài giải:

 

sữ dụng đánh giá sau:
$\sqrt{(b+2)(b^{2}-b+2)}\leq \frac{b+2+b^{2}-b+2}{2}\leq \frac{b^{2}+4}{2}$
ta đưa bài toán về chứng minh
$\sum \frac{a}{b^{2}+4}\geq \frac{3}{4}$
ta có $\frac{a}{b^{2}+4}=\frac{a}{4}-\frac{ab^{2}}{4(b^{2}+4)}\geq \frac{a}{4}-\frac{ab^{2}}{4.4b}\geq \frac{a}{4}-\frac{ab}{16}$
làm tương tự 2 phânt thức kia kết hợp với bđt $ab+bc+ca\leq \frac{(a+b+c)^{2}}{3}\leq 12$ là ra

 

Bài 105:

Cho a,b thuộc [0;1]. CMR:
$a+b+\frac{(a-1)(b-1)}{ab+1}\geq 1+\frac{a^2b+ab^2}{2(ab+1)}$


(thpt YD2)

Bài giải:

 

 

Ta có bất đẳng thức tương đương với ^^~
$$a+b-\frac{ab(a+b)}{2(ab+1)}\geq 1-\frac{(a-1)(b-1)}{ab+1}$$
$$\Leftrightarrow (a+b).\left(1-\frac{ab}{2(ab+1)}\right)\geq \frac{a+b}{ab+1}$$
$$\Leftrightarrow \frac{ab+2}{2(ab+1)}\geq \frac{1}{ab+1}$$
$$\Leftrightarrow \frac{ab+2}{2}\geq 1$$
$$\Leftrightarrow ab\geq 0$$
Bất đẳng thức cuối luôn đúng nên ta có điều phải chứng minh.Đẳng thức xảy ra khi $a,b$ có 1 số bằng 0 ' '~

 

 

Bài 106:
các bạn mình là mem mới. xin các bạn giúp mình bài này với,minh rat can su tro giup cua cac ban;
chứng minh bất dẳng thúc này đúng với mọi tam giác ABC:
$2\sqrt{2}\left ( \sin \dfrac{A}{2}+\sin \dfrac{B}{2}+\sin \dfrac{C}{2} \right )> \cos \dfrac{A-B}{\sqrt{15}}+\cos \dfrac{B-C}{\sqrt{15}}+\cos \dfrac{C-A}{\sqrt{15}}$

Bài giải:

 

 

Ta có $cos\frac{C}{2}\left ( sin\frac{A}{2}+sin\frac{B}{2} \right )$;

$cos\frac{B}{2}\left ( sin\frac{C}{2}+sin\frac{A}{2} \right )$;
$cos\frac{A}{2}\left ( sin\frac{B}{2}+sin\frac{C}{2} \right )$.
Cộng vế với vế các bất đảng thức trên ta được
$sin\frac{A+B}{2}+sin\frac{B+C}{2}+sin\frac{C+A}{2}$$<2\left ( sin\frac{A}{2}+sin\frac{B}{2}+sin\frac{C}{2} \right )$.
hay $cos\frac{A}{2}+cos\frac{B}{2}+cos\frac{C}{2}<2\left ( sin\frac{A}{2}+sin\frac{B}{2}+sin\frac{C}{2} \right )$.
Đến đây chỉ cần chứng minh $\sqrt{2}\left ( cos\frac{A}{2}+cos\frac{B}{2}+cos\frac{C}{2} \right )>\sum cos\frac{A-B}{\sqrt{15}}$ là xong.
Ta có $cos\frac{A}{2}+cos\frac{B}{2}=2cos\frac{A+B}{4}cos\frac{A-B}{4}>2.cos\frac{\pi }{4}cos\frac{A-B}{4}=\sqrt{2}cos\frac{A-B}{4}$.
Mặt khác $\frac{\left | A-B \right |}{4}<\frac{\left | A-B \right |}{\sqrt{15}}<\frac{\pi }{2}$.
Suy ra $cos\frac{A}{2}+cos\frac{B}{2}>\sqrt{2}cos\frac{A-B}{\sqrt{15}}$. (*)
Cộng vế với vế các BĐT tương tự (*) ta thu được đpcm.$\square$
 

Bài 107

cho a,b,c >0 thoả mãn:a+b+c=3.CMR:
$\sum a^{2}+\frac{ab+bc+ac}{a^{2}b+b^{2}c+c^{2}a}\geq 4$

 

Bài giải:

 

BĐT tương đương với
$(a)^2-2\sum{ab}+\dfrac{3\sum{ab}}{(\sum{a})(\sum{a^2b})}\geq 9-2\sum{ab})+\dfrac{3\sum{ab}}{3\sum{a^2}}= 9-2q+\dfrac{q}{9-2q}$

Cần chứng minh $9 - 2q +\dfrac{q}{9-2q} \geq 4 \Leftrightarrow (q-3,75)(q-3)\geq 0$ (đúng vì $q \leq 3$)

 

 

Bài 108:
Cho $a,b,c$ là $3$ cạnh tam giác.CMR: \[\sum \frac{(b+c-a)^{4}}{a(a+b-c)}\geq ab+bc+ac\]

 

 

Bài giải:

đặt:
$b+c-a=x$
$c+a-b=y$
$a+b-c=z$
$\Rightarrow a=\frac{y+z}{2};b=\frac{z+x}{2};c=\frac{x+y}{2}$

BDT cần cm tuơng đuơng
$\sum \frac{x^{4}}{\frac{(y+z)z}{2}}\geq \sum \frac{(x+y)(y+z)}{4}$
ta có:
$\sum \frac{x^{4}}{\frac{(y+z)z}{2}}\geq \frac{(\sum x^{2}) ^{2}}{\frac{1}{2}\sum (z^{2}+yz)}$

$\sum xy\leq \sum x^{2}\Rightarrow VT\geq \sum x^{2}$
ta can cm
$\sum x^{2}\geq \sum \frac{1}{4}\sum (x+y)(y+z)$
hay $\sum x^{2}\geq \sum xy$ ( hien nhien)
dau "=" xay ra khi x=y=z hay a=b=c

 

 

Bài 109: Chứng minh với mọi a,b,c không âm,không có 2 số nào đồng thời bằng không 
$\frac{1}{a^2+bc}+\frac{1}{b^2+ca}+\frac{1}{c^2+ab}\geq \frac{3}{ab+bc+ca}$

Đây là 1 bài toán khá hay. Nó là hệ quả của 1 BDT của anh Phạm Kim Hùng
Giải như sau:
Bất đẳng thức đã cho tương đương với:
$\sum \frac{a(b+c)}{a^2+bc}+\sum \frac{bc}{a^2+bc}\geq 3$

+++)
Theo Cauchy-Schwarz thì $\sum \frac{bc}{a^2+bc}\geq\frac{(ab+ac+bc)^2}{abc(a+b+c)+\sum b^2c^2}\geq 1$
+++)
Ta chưng minh:
$\sum \frac{a(b+c)}{a^2+bc}\geq 2$ (*)
Xét khai triển sau:
$(a-b)^2(b-c)^2(c-a)^2\geq 0\Leftrightarrow \sum a^4(b^2+c^2)+2abc\sum a^2(b+c)\geq 2\sum a^3b^3+6a^2b^2c^2+2abc\sum a^3$
(*)$\Leftrightarrow \sum a(b+c)(c^2+ab)(b^2+ac)\geq 2(a^2+bc)(b^2+ac)(c^2+ab)\Leftrightarrow 2abc\sum a^2(b+c)+\sum a^4(b^2+c^2)+abc\sum a^2(b+c)\geq 4a^2b^2c^2+2\sum a^3b^3+2abc\sum a^3$
Kết hợp khai triển trên và $2a^2b^2c^2+abc\sum a^2(b+c)\geq 0$ ta có điều phải chứng minh
Đẳng thức xảy ra khi và chỉ khi a=b,c=0 hoặc các hoán vị
Chứng minh của ta được hoàn tất.

 

Bài 120.Chứng minh với mọi a,b,c không âm 
$\frac{1}{\frac{1}{a+1}+\frac{1}{b+1}+\frac{1}{c+1}}-\frac{1}{\frac{1}{a}+\frac{1}{b}+\frac{1}{c}}\geq \frac{1}{3}$

 

 

 

Giải như sau:
Nhân tung lên ta được đpcm tương đương với:
$\frac{(a+1)(b+1)(c+1)}{ab+ac+bc+2(a+b+c)+3}-\frac{abc}{ac+ac+bc}\geq \frac{1}{3}$
Đến đây đặt x=a+b+c,y=ab+ac+bc,z=abc ta có điều trên tương đương với:$3xy+2y^2\geq 6xz+9z$
Điều này đúng do $(ac+ab+bc)^2\geq 3abc(a+b+c);(a+b+c)(ab+ac+bc)\geq 9abc$( theo AM-GM)
Vậy ta có đpcm
Đẳng thức xảy ra khi và chỉ khi a=b=c

 

Bài 121:

Cho $\bigtriangleup ABC$$\bigtriangleup ABC$ và M nằm trong tam giác. BC=a,CA=b,Ab=c
Gọi khoảng cách từ M=>BC là x, M=>AC là y,M=>AB là z.
cmr $\sqrt{x}+\sqrt{y}+\sqrt{z}\leq \sqrt{\frac{a^{2}+b^{2}+c^{2}}{2R}}$ ( R là bán kính đường tròn ngoại tiếp) và $\sqrt{x}+\sqrt{y}+\sqrt{z}\leq \sqrt{\frac{(a^{2}+b^{2}+c^{2})2S}{abc}}$ (S là diện tích tam giác) có giống nhau không vâyhj(Nếu giống biến đổi kiểu gì vậy?

 

Bài giải:

 

ĐẲng thức quen thuộc $R=\frac{abc}{4S}$ mà bạn 
Ta có điều phải chứng minh tương đương:
$$\left(\sqrt{x}+\sqrt{y}+\sqrt{z}\right)^2\leq \frac{(a^{2}+b^{2}+c^{2})2S}{abc}$$
Sử dụng bất đẳng thức $Cauchy-Schwarz$ và để ý $ax+by+cz=2S$ ta có:
$$\left(\sqrt{x}+\sqrt{y}+\sqrt{z}\right)^2\leq (ax+by+cz).\left(\frac{1}{a}+\frac{1}{b}+\frac{1}{c}\right)$$
$$=2S.\frac{ab+bc+ca}{abc}\leq \frac{(a^{2}+b^{2}+c^{2})2S}{abc}$$
Kết thúc chứng minh.Đẳng thức xảy ra tại $a=b=c$,tam giác của ta là tam giác đều $\square$

 

 

Bài 122:
Cho $a,b> 0, a+b=\frac{1}{2}. CM: \frac{1}{a^2+b^2}+\frac{10}{\sqrt{a}}+\frac{10}{\sqrt{b}}\geq 48.$

Bài giải:

 

 

Bạn đừng vội quá. Có gợi ý ch0 bạn đây 
$\bullet$ Áp dụng liên tiếp bất đẳng thức $AM-GM$ ta có:
$$\frac{1}{a^2+b^2}+\frac{4}{\sqrt{a}}+\frac{4}{\sqrt{a}}+\frac{4}{\sqrt{b}}+\frac{4}{\sqrt{b}}\geq 5.\sqrt[5]{\frac{4^4}{(a^2+b^2)ab}}$$
$$=5.\sqrt[5]{\frac{2^{11}}{8(a^2+b^2)ab}}= 5.\sqrt[5]{\frac{2^{11}}{4(a^2+b^2)2ab}}$$
$$\geq 5.\sqrt[5]{\frac{2^{11}}{(a+b)^4}}=5.\sqrt[5]{2^{15}}=40$$
$\bullet$ Và mặt khác ta lại có:
$$\frac{2}{\sqrt{a}}+\frac{2}{\sqrt{b}}\geq \frac{4}{\sqrt[4]{ab}}\geq \frac{4\sqrt{2}}{\sqrt{a+b}}$$
$$=4\sqrt{2}.\sqrt{2}=8$$
Cộng 2 bất đẳng thức cùng chiều trên ta có điều phải chứng minh thôi :") $\square$

Bài 123:

cho a,b,c>0 thoả mãn:a+b+c=$\frac{1}{2}$

,Tìm GTLN:$\sum \sqrt{\frac{(a+b)(b+c)}{(a+b)(b+c)+a+c}}$

 

Bài giải:

 

Đặt $x=a+b;y=b+c;z=c+a$
Có:
$P=\sum \sqrt{\frac{xy}{xy+z}}=\sum \sqrt{\frac{xy}{(x+z)(z+y)}}\leq\frac{1}{2}\sum (\frac{x}{x+z}+\frac{y}{z+y})=\frac{3}{2}\blacksquare$
$"=" <=>x=y=z=\frac{1}{6}$

Bài 124:

 

$Cho a,b,c> 0, CM:\sqrt{\frac{ab+bc+ca}{a^2+b^2+c^2}}+\frac{a+b+c}{\sqrt[3]{abc}}\geq 4$

Bài giải:

 

 Áp dụng trực tiếp AM-GM thôi:
$\sqrt{\frac{ab+bc+ca}{a^2+b^2+c^2}}+\frac{a+b+c}{\sqrt[3]{abc}}\geq 4\sqrt[4]{\left ( \frac{a+b+c}{3\sqrt[3]{abc}} \right )^3.\sqrt{\frac{ab+bc+ca}{a^2+b^2+c^2}}}$
Nên ta cần chứng minh:
$\left ( \frac{a+b+c}{3\sqrt[3]{abc}} \right )^3.\sqrt{\frac{ab+bc+ca}{a^2+b^2+c^2}}\geq 1\Leftrightarrow \frac{(a+b+c)^6}{(3^6a^2b^2c^2}.\frac{ab+bc+ca}{a^2+b^2+c^2}\geq 1$
Ta có:
$\frac{(a+b+c)^6}{3^6a^2b^2c^2}.\frac{ab+bc+ca}{a^2+b^2+c^2}\geq \frac{(a+b+c)^6(ab+bc+ca)}{3^3(ab+bc+ca)^3(a^2+b^2+c^2)}$
$=\frac{\left ( \frac{a^2+b^2+c^2+ab+bc+ca+ab+bc+ca}{3} \right )^3}{(a^2+b^2+c^2)(ab+bc+ca)^2}\geq 1$

Bài 125:

 

Cho x,y thuộc [-1;1]. CMR:
$\sqrt{x^2+(y-1)^2}+\sqrt{x^2+(y+1)^2}+\sqrt{(x-1)^2+y^2}+\sqrt{(x+1)^2+y^2}\geq 4$

Bài giải:

Sr nhìn không kĩ, Min chỉ c/m được $\ge 2\sqrt{2}$...
----

Tổng quát $x,y\in \mathbb{R}$ thì BĐT vẫn đúng:
Dự đoán điểm rơi ở $x=y=0$ nên ta có các đánh giá sau:
$$\sqrt{x^2+(y-1)^2}\ge \sqrt{(y-1)^2}=|1-y|\\ \sqrt{x^2+(y+1)^2}\ge \sqrt{(y+1)^2}=|y+1|\\ \sqrt{(x-1)^2+y^2}\ge \sqrt{(x-1)^2}=|1-x|\\\sqrt{(x+1)^2+y^2}\ge \sqrt{(x+1)^2}=|x+1|$$
Từ các điều trên suy ra:
$$LHS\ge |1-y|+|y+1|+|1-x|+|x+1|\ge |1-y+y+1+1-x+x+1|=4=RHS$$
Dấu bằng xảy ra khi $x=y=0\ \square$
----
P/s: Cái giả thiết làm được gì cho đời nhỉ?

 

Bài 126:

 

Cho $a,b,c>0$. Chứng minh rằng: \[ \frac{1}{2a+b}+\frac{1}{2b+c}+\frac{1}{2c+a}\geq \frac{2}{\sqrt[3]{(a+b)(b+c)(c+a)}}\]

 

Bài giải:

Bài này không khó nhưng phải tinh ý chút  :)
Nhân thêm 2 vế với $a+b+c$ và biến đổi 1 chút ta sẽ có:
$$\sum_{cyc}\frac{c}{2a+b}+\frac{1}{2}\sum_{cyc}\frac{b}{2a+b}+\frac{3}{2} \ge \frac{2(a+b+c)}{\sqrt[3]{(a+b)(b+c)(c+a)}}$$
Theo C-S:
$$\sum_{cyc}\frac{c}{2a+b} \ge \frac{(a+b+c)^2}{3(ab+bc+ca)}$$
$$\sum_{cyc}\frac{b}{2a+b} \ge \frac{(a+b+c)^2}{a^2+b^2+c^2+2(ab+bc+ca)}=1$$
Vậy ta cần chứng minh:
$$\frac{(a+b+c)^2}{3(ab+bc+ca)}+2 \ge \frac{2(a+b+c)}{\sqrt[3]{(a+b)(b+c)(c+a)}}$$
Theo AM-GM:
$$\frac{(a+b+c)^2}{3(ab+bc+ca)}+2 \ge 3\sqrt[3]{\frac{(a+b+c)^2}{3(ab+bc+ca)}}$$
Bài toán sẽ được giải quyết nếu ta chứng minh :
$$3\sqrt[3]{\frac{(a+b+c)^2}{3(ab+bc+ca)}} \ge \frac{2(a+b+c)}{\sqrt[3]{(a+b)(b+c)(c+a)}}$$
Tương đương với:
$$(a+b)(b+c)(c+a) \ge \frac{8}{9}(ab+bc+ca)(a+b+c) \iff c(a-b)^2+a(b-c)^2+b(c-a)^2 ]ge 0$$
Luôn đúng.Đẳng thức xảy ra khi $a=b=c$.

Bài 127:

 

Chứng minh rằng: $\frac{3(a^4+b^4+c^4)}{(a^2+b^2+c^2)^2}+\frac{ab+bc+ca}{a^2+b^2+c^2}\geq 2.$

 

Bài giải:

$3\frac{\sum x^{4}}{(\sum x^{2})}+\frac{\sum xy}{\sum x^{2}}\geq 2 \Leftrightarrow 3(\sum x^{4})+\sum xy\sum x^{2}\geq 2((\sum x^{2})^{2})$

hay

$x^{4}+y^{4}+z^{4}+xy(x^{2}+y^{2})+yz(y^{2}+z^{2})+zx(z^{2}+x^{2})\geq 4(x^{2}y^{2}+y^{2}z^{2}+z^{2}x^{2} )-xyz(x+y+z)$

$xyz(x+y+z)\leq x^{2}y^{2}+y^{2}z^{2}+z^{2}x^{2}\Rightarrow -xyz(x+y+z)\geq -(x^{2}y^{2}+y^{2}z^{2}+z^{2}x^{2})$


nen ta can chung minh

$x^{4}+y^{4}+z^{4}+xy(x^{2}+y^{2})+yz(y^{2}+z^{2})+zx(z^{2}+x^{2})\geq 3(x^{2}y^{2}+y^{2}z^{2}+z^{2}x^{2} )$

ap dung AM-GM ta có

$x^{4}+y^{4}+z^{4}\geq x^{2}y^{2}+y^{2}z^{2}+z^{2}x^{2}$



$xy(x^{2}+y^{2})+yz(y^{2}+z^{2})+zx(z^{2}+x^{2})\geq 2(x^{2}y^{2}+y^{2}z^{2}+z^{2}x^{2})$

cong vo theo ve 2 bdt tren ta co dpcm
dau "=" khi $x=y=z$

 

Bài 128:

Với mọi a,b,c>0chứng minh rằng:
$\frac{(a+b+c)^3}{a^2b+b^2c+c^2a+3abc}\geq \frac{9}{2}$

 

 

Bài giải:

$a^2b+b^2c+c^2a+abc\leq \frac{4}{27}(a+b+c)^3$ 

 

Không mất tính tổng quát ta giả sử b nằm giữa c và a.
suy ra $(b-a)(b-c)\leq 0\rightarrow c(b-a)(b-c)\leq 0$
suy ra $b^2c+c^2a\leq bc^2+abc$
$\Leftrightarrow b^2c+c^2a+a^2b+abc\leq bc^2+a^2b+2abc\leq \frac{1}{2}2b(a+c)^2$
$\leq \frac{1}{2}\left [ \frac{(2b+a+c+a+c)}{3} \right ]^3$$=\frac{1}{2}\frac{\left [ 2(a+b+c) \right ]^3}{27}=4$
dấu đẳng thức xảy ra khi và chỉ khi a=b=c=1 hoặc a=2,b=1,c=0.

 

và $2abc\leq \frac{2}{27}(a+b+c)^3$
nên $\frac{(a+b+c)^3}{a^2b+b^2c+c^2a+3abc}\geq \frac{1}{\frac{4}{27}+\frac{2}{27}}=\frac{9}{2}$

 

 

 

 

 

 




#436988 0,99... = 1 ?

Gửi bởi phanquockhanh trong 21-07-2013 - 22:01

Như mọi người đã biết rồi đó:

 

 

Theo quy tắc tính cấp số nhân lùi vô hạn thì:

 

0.33333333333333333333333333333333333333333= 1phân3

 

=> 0.99999999999999999999999999999999= 3*0.333333333333333333333333333333333=3*1phan3=1 (vô lý )

 

Ai biết thì xin chỉ dùm. Mình xin được cảm ơn 

Bạn phải viết như thế này mới đúng nhé :$ 0,333...=0,(3)=\frac{1}{3} $.

 Việc ghi 0,(9) = 1 là phù hợp theo cách tính tổng của một cấp số nhân lùi vô hạn (Sách giáo khoa đã đề cập )

===============

Bạn có thể chứng minh thêm đẳng thức sau để hiểu rõ hơn về những điều ấy :

$\frac{\pi}{2}=\frac{1}{\sqrt{\frac{1}{2}}.\sqrt{\frac{1}{2}+\frac{1}{2}\sqrt{\frac{1}{2}}}.\sqrt{\frac{1}{2}+\frac{1}{2}\sqrt{\frac{1}{2}+\frac{1}{2}\sqrt{\frac{1}{2}...}}}}$




#435260 $ P= \frac{{{x^2}}}{{{...

Gửi bởi phanquockhanh trong 14-07-2013 - 18:08

Cho các số thực dương $x,y,z$  thỏa mãn điều kiện $2\left( {9{z^2} + 16{y^2}} \right) = \left( {3z + 4y} \right)xyz$ . Tìm giá trị nhỏ nhất của biểu thức : 
$P = \frac{{{x^2}}}{{{x^2} + 2}} + \frac{{{y^2}}}{{{y^2} + 3}} + \frac{{{z^2}}}{{{z^2} + 4}} + \frac{{5xyz}}{{\left( {x + 2} \right)\left( {y + 3} \right)\left( {z + 4} \right)}}$
 

 




#435068 $P=\frac{2}{x^2+1} - \frac{2}...

Gửi bởi phanquockhanh trong 13-07-2013 - 19:09

Cho $x,y,z >0 : xyz+x+z=y$ . Tìm giá trị lớn nhất của biểu thức : $P=\frac{2}{x^2+1} - \frac{2}{y^2+1} -\frac{4z}{\sqrt{z^2+1}}+\frac{3z}{(z^2+1).\sqrt{z^2+1}}$

(Trích đề thi thử số 2 – THTT)

 

 




#435064 $\sum \frac{a^2}{\sqrt{2a^2 +ab +b^2...

Gửi bởi phanquockhanh trong 13-07-2013 - 18:55

Cho a,b,c là các số thực dương.
Chứng minh rằng:

\[\dfrac{{{a^2}}}{{\sqrt {2{a^2} + ab + {b^2}} }} + \dfrac{{{b^2}}}{{\sqrt {2{b^2} + bc + {c^2}} }} + \dfrac{{{c^2}}}{{\sqrt {2{c^2} + ca + {a^2}} }} \ge \dfrac{{a + b + c}}{2}\]

 




#434608 $P= \frac{xy +yz +zx}{x+y+z} +\frac{2...

Gửi bởi phanquockhanh trong 11-07-2013 - 21:44

Cho các  số thực  $x;y;z >0$ ;$x^2 +y^2 +z^2 =\frac{1}{3}$.Tìm giá trị nhỏ nhất của biểu thức :

$P= \frac{xy +yz +zx}{x+y+z} +\frac{2(x+y+z)}{9(xy+yz+zx)}$




#434607 $P= 5x^2 +y^2 +2z^2+\frac{5}{16x^2 + z^2 +6xy+12yz...

Gửi bởi phanquockhanh trong 11-07-2013 - 21:39

Cho $x;y;z> 0$ thỏa $xy+yz +zx =1$.Tìm giá trị nhỏ nhất của biểu thức:

$P= 5x^2 +y^2 +2z^2+\frac{5}{16x^2 + z^2+6xy+12yz}$ 




#434606 $P= \frac{(z+z\sqrt{xy})^2}{(x+y)(z^2...

Gửi bởi phanquockhanh trong 11-07-2013 - 21:34

Cho các số thực  $x,y,z >0$ thỏa mãn: $x =y+z +xyz$
Tìm giá trị lớn nhất của biểu thức :
$P= \frac{(z+z\sqrt{xy})^2}{(x+y)(z^2 +1)} +\frac{2z}{(z^2 +1)\sqrt{z^2 +1}}$
 

 




#432917 Tổng hợp các bài BĐT - GTLN GTNN thi thử đại học

Gửi bởi phanquockhanh trong 04-07-2013 - 22:05

Bài 22: Cho $a,b,c$ là $3$ số dương thỏa mãn: $a+b-c \geq 0, b+c-a \geq 0, c+a-b \geq 0, (a+b+c)^2=4(ab+bc+ca-1)$. Tìm giá trị nhỏ nhất của biểu thức: $S= \sqrt{ \dfrac{b+a}{c}-1}+ \sqrt{ \dfrac{a+c}{b}-1}+ \sqrt{ \dfrac{c+b}{a}-1}+ \dfrac{2 \sqrt{2}}{ \sqrt{a^2+b^2+c^2-2}}$.

 (Đề thi thử trường THPT Nguyến Khuyến)